9

I came across this integral in some work. So, I would like to ask:

QUESTION. Can you evaluate this integral with proofs? $$\int_0^1\frac{\log x\cdot\log(x+2)}{x+1}\,dx.$$

GH from MO
  • 98,751
T. Amdeberhan
  • 41,802
  • 1
    Wolfram Alpha gives a closed-form solution. – Carl-Fredrik Nyberg Brodda Mar 01 '21 at 14:36
  • 8
    I think it is bad form silently to edit your question to ask for proofs, after @CarloBeenakker already gave an answer to the original version that did not request proofs. (That doesn't mean you can't ask for proofs, just that I think it's polite to point out that that is a new request.) – LSpice Mar 01 '21 at 15:11
  • 3
    @Carlo Beenakker and all: I apologize for the edit. I was always under the impression that when you evaluate something, you also give a proof. But, that was not happening earlier, so to clarify I added "with proof". – T. Amdeberhan Mar 01 '21 at 15:13
  • This is actually non-trivial: WolframAlpha's closed form for the definite integral has no proof, and Mathematica's closed form for the indefinite integral is complex and not easy to use at 0 or 1. –  Mar 01 '21 at 16:21
  • Is there a nice expression for some $f(x)$ with $f'(x)=\log(x)\log(x+2)/(x+1)$ which is obviously real on $0<x<1$? I have not been able to get one out of Mathematica. –  Mar 01 '21 at 17:00
  • 6
    @LSpice: I do not think that answers without proofs for such questions belong on MathOverflow in the first place, so editing the question to indicate this implicitly understood convention is not bad form. – Dmitri Pavlov Mar 01 '21 at 17:13
  • 3
    I'd split the difference between LSpice and Dmitri by saying that answers which are just numbers you got from computer software are okay, in so far as they can certainly be helpful for getting a complete solution, but that asking for a better answer with a true proof is totally reasonable too. – Sam Hopkins Mar 01 '21 at 17:47
  • For what it's worth, $\int_0^1 (\log x)^2/(x+1) dx = \tfrac32 \zeta(3)$ is easily found by series expansion of the denominator. – Mateusz Kwaśnicki Mar 01 '21 at 20:44
  • Maybe we should talk about the fact this is integral of 2 ln(f) / f' ? – Richard Birkett Mar 01 '21 at 23:48
  • 7
    This appears to be a duplicate of this MSE question. – Timothy Budd Mar 02 '21 at 07:42
  • 1
    @RichardBirkett Please note that the two logs are multiplied, not added^^ – Wolfgang Mar 02 '21 at 09:36

2 Answers2

11

For $t\in(0,1]$, let \begin{equation} I(t):=\int_0^1\frac{\log(x)\,\log(1+t(x+1))}{x+1}\,dx \end{equation} (so that the integral in question is $I(1)$),
\begin{equation} \begin{aligned} J(t)&:=-\text{Li}_3(2+1/t)+\text{Li}_3(-2 t-1)+\text{Li}_3(t+1) \\ &-\text{Li}_3(2t+1) +\text{Li}_2(2+1/t) (\log (2 t+1)+i \pi ) \\ &+\text{Li}_2(-2 t-1)(-\log (2 t+1)-i \pi ) \\ &+\text{Li}_2(t+1) (-\log (t+1)-i \pi )+\text{Li}_2(2 t+1) (\log (2 t+1)+i \pi ) \\ &+\frac{1}{6} (\log ^3(t)+(-3 \log ^2(t+1)-6 i \pi \log (t+1)+4 \pi ^2) \log (t) \\ &+3 \pi (-i \log ^2(t+1)+2 i \log ^2(2 t+1)+2 \pi \log (t+1)-4 \pi \log (2 t+1))) \\ &+\frac{3 \zeta (3)}{4}-\frac{5 i \pi ^3}{12}, \end{aligned} \end{equation} \begin{equation} \begin{aligned} I_1(t)&:=6tI'(t)=6t\int_0^1\frac{\log(x)}{1+t(x+1)}\,dx \\ & =6 \text{Li}_2\left(\frac{t+1}{2 t+1}\right)-3 \log ^2(t+1)+3 \log ^2(2 t+1) \\ &+6 \log (t) \log \left(\frac{t+1}{2 t+1}\right)-\pi ^2, \end{aligned} \end{equation} \begin{equation} \begin{aligned} J_1(t)&:=6tJ'(t)=6 \text{Li}_2\left(2+1/t\right)+3 \log ^2(t)-3 \log ^2(t+1) \\ &-6 (\log (2 t+1)+i \pi ) \log (t)+6 \log (t+1) \log (2 t+1) \\ &+6 i \pi \log (2 t+1)-2 \pi ^2. \end{aligned} \end{equation} Then $I'_1=J'_1$ and $I_1(0+)=J_1(0+)$, so that $I_1=J_1$, and hence $I'=J'$. Also, $I(0)=I(0+)=0=J(0+)$, so that $I=J$, and the integral in question is \begin{equation} \begin{aligned} I(1)&=J(1)=\text{Li}_3(-3)-2 \text{Li}_3(3)+i \pi \left(-\text{Li}_2(-3)+2 \text{Li}_2(3)+\log ^2(3)\right) \\ &-\text{Li}_2(-3) \log (3)+\text{Li}_2(3) \log (9)+\frac{13 \zeta (3)}{8}-\frac{2 i \pi ^3}{3}-\pi ^2 \log (9) \\ &=-0.651114\dots. \end{aligned} \end{equation}

Iosif Pinelis
  • 116,648
7

$$\int_0^1\frac{\ln x\cdot\ln(x+2)}{x+1}\,dx=$$ $$=\text{Li}_3\left(-\tfrac{1}{3}\right)-2 \,\text{Li}_3\left(\tfrac{1}{3}\right)+\tfrac{1}{2} \ln 3\left[ \text{Li}_2\left(\tfrac{1}{9}\right)-6\, \text{Li}_2\left(\tfrac{1}{3}\right) -\tfrac{2}{3} \ln ^2 3\right]+\tfrac{13}{8} \zeta (3).$$ I checked that this combination of polylog's evaluates to $-0.651114$, equal to a numerical evaluation of the integral.

Update: As Timothy Budd pointed out, that this combination of polylog's simplifies to $-\frac{13}{24}\zeta(3)$ is proven by Przemo at MSE.
The identities that enable this simplification are $$\text{Li}_3\left(-\tfrac{1}{3}\right)-2 \,\text{Li}_3\left(\tfrac{1}{3}\right) = -\tfrac{1}{6} \ln^3 3 + \tfrac{1}{6}\pi^2 \ln 3 - \tfrac{13}{6} \zeta(3),$$ $$\text{Li}_2(\tfrac{1}{9})=2\,\text{Li}_2(-\tfrac{1}{3})+2\,\text{Li}_2(\tfrac{1}{3}),$$ $$2\text{Li}_2\left(-\tfrac{1}{3}\right)-4 \,\text{Li}_2\left(\tfrac{1}{3}\right) = \ln^2 3 -\tfrac{1}{3}\pi^2 .$$

Carlo Beenakker
  • 177,695
  • 2
    Shouldn't you be able to eliminate the imaginary parts? – user174996 Mar 01 '21 at 14:40
  • @CarloBeenakker: is there an even simpler answer and proof? – T. Amdeberhan Mar 01 '21 at 14:50
  • 1
    I computed numerically that the value is $-\frac{13\zeta(3)}{24}$, so it seems. Hope this shades some light into the problem. – T. Amdeberhan Mar 01 '21 at 19:14
  • 6
    @T.Amdeberhan: On 18 April 2017 you knew already (with proof) that the integral equals $-\frac{13\zeta(3)}{24}$. See your own response at https://mathoverflow.net/questions/267485/an-interesting-integral-expression-for-pin So it is not clear why you say that "I computed numerically etc." Of course it is legitimate and useful to ask for simpler proofs. – GH from MO Mar 02 '21 at 10:54
  • Do you have a proof of the first display (other than what we can learn from Iosif Pinelis's answer and https://math.stackexchange.com/questions/1344455/proving-that-int-01-frac-log-left-frac1t-right-log-t2t1-d)? I mean, where did you get the first display from? – GH from MO Mar 02 '21 at 10:58
  • 2
    @GHfromMO: Thank you, this is helpful. I completely forgot about this encounter. – T. Amdeberhan Mar 02 '21 at 19:54